-
Notifications
You must be signed in to change notification settings - Fork 18
/
Copy pathCMC.tex
executable file
·300 lines (265 loc) · 13.1 KB
/
CMC.tex
1
2
3
4
5
6
7
8
9
10
11
12
13
14
15
16
17
18
19
20
21
22
23
24
25
26
27
28
29
30
31
32
33
34
35
36
37
38
39
40
41
42
43
44
45
46
47
48
49
50
51
52
53
54
55
56
57
58
59
60
61
62
63
64
65
66
67
68
69
70
71
72
73
74
75
76
77
78
79
80
81
82
83
84
85
86
87
88
89
90
91
92
93
94
95
96
97
98
99
100
101
102
103
104
105
106
107
108
109
110
111
112
113
114
115
116
117
118
119
120
121
122
123
124
125
126
127
128
129
130
131
132
133
134
135
136
137
138
139
140
141
142
143
144
145
146
147
148
149
150
151
152
153
154
155
156
157
158
159
160
161
162
163
164
165
166
167
168
169
170
171
172
173
174
175
176
177
178
179
180
181
182
183
184
185
186
187
188
189
190
191
192
193
194
195
196
197
198
199
200
201
202
203
204
205
206
207
208
209
210
211
212
213
214
215
216
217
218
219
220
221
222
223
224
225
226
227
228
229
230
231
232
233
234
235
236
237
238
239
240
241
242
243
244
245
246
247
248
249
250
251
252
253
254
255
256
257
258
259
260
261
262
263
264
265
266
267
268
269
270
271
272
273
274
275
276
277
278
279
280
281
282
283
284
285
286
287
288
289
290
291
292
293
294
295
296
297
298
299
300
\documentclass[hideanswer=false,
enfont=newtxtext,
zhfont=empty,
mathfont=newtxmath,
]{cmcthesis}
\cmcthesissetup{% key = value 设置处
%
}
\input{settings.tex}
\begin{document}
\cmcthesistitle{
name = 第十届全国大学生数学竞赛\LaTeX 模板 ,
subname = 非数类参考解析,
date = 2018年10月27号 \thinspace 9:00 - 11:30 ,
author = (模板制作者:八一与酸奶) ,
motto = 微信公众号:八一考研数学竞赛 ,
}
\addvspace{1\bigskipamount}
\ws{填空题}{( \textbf{本题满分24分,每题6分})\\}\\\\
\wq 设 $\alpha\in\left(0,1\right),\textrm{则} \lim_{n\rightarrow +\infty}\left(\left(n+1\right)^{\alpha}-n^{\alpha}\right)=$\underline{\hspace{3em}}.\\
\begin{answer}
\begin{solution}
等价无穷小$\left(1+x\right)^{\alpha}-1\backsim\alpha x$,得
\[
\lim_{n\rightarrow\infty}\left(\left(n+1\right)^{\alpha}-n^{\alpha}\right)=\lim_{n\rightarrow\infty}n^{\alpha}\left(\left(1+1/n\right)^{\alpha}-1\right)=\lim_{n\rightarrow\infty}n^{\alpha}\times\frac{\alpha}{n}=0
\]
\end{solution}
\end{answer}
\wq $\textrm{若曲线}y=f\left(x\right)\textrm{是由}\left\{\begin{array}{l}
x=t+\cos t\\
\mathrm{e}^y+ty+\sin t=1\\
\end{array}\right.\textrm{确定,则此曲线在}t=0$ 对应点处的\\
切线方程为\underline{\hspace{3em}}.\\
\begin{answer}
\begin{solution}
易知$t=0$处上的曲线为点$(1,0)$,即方程组对$t$求导得
\[
\frac{\mathrm{d}x}{\mathrm{d}t}=1-\sin t\textbf{,}\frac{\mathrm{d}y}{\mathrm{d}t}=-\frac{y+\cos t}{\mathrm{e}^y+t}
\]
\[
\Rightarrow\frac{\mathrm{d}y}{\mathrm{d}t}=\frac{\mathrm{d}y}{\mathrm{d}t}/\frac{\mathrm{d}x}{\mathrm{d}t}=-\frac{y+\cos t}{\left(\mathrm{e}^y+1\right)\left(1-\sin t\right)}\Rightarrow\left. \frac{\mathrm{d}y}{\mathrm{d}x}\right| _{t=0}=-1
\]
故曲线在$t=0$对应点处的切线方程为$x+y-1=0$.
\end{solution}
\end{answer}
\wq $\int{\frac{\ln\left(x+\sqrt{1+x^2}\right)}{\left(1+x^2\right)^{\frac{3}{2}}}}\mathrm{d}x=$\underline{\hspace{3em}}.\\
\begin{answer}
\begin{solution}
简单的凑微分,如下
\begin{align*}
\int{\frac{\ln\left(x+\sqrt{1+x^2}\right)}{\left(1+x^2\right)^{\frac{3}{2}}}}\mathrm{d}x&=\int{\ln\left(x+\sqrt{1+x^2}\right)}\mathrm{d}\left(\frac{x}{\sqrt{1+x^2}}\right)\\
&=\frac{x}{\sqrt{1+x^2}}\ln\left(x+\sqrt{1+x^2}\right)-\int{\frac{x}{1+x^2}\mathrm{d}x}\\
&=\frac{x}{\sqrt{1+x^2}}\ln\left(x+\sqrt{1+x^2}\right)-\frac{1}{2}\ln\left(1+x^2\right)+C
\end{align*}
\end{solution}
\end{answer}
\wq $\lim_{x\rightarrow 0}\frac{1-\cos x\sqrt{\cos 2x}\sqrt[3]{\cos 3x}}{x^2}=$\underline{\hspace{3em}}.\\
\begin{answer}
\begin{solution}
由$\cos x= 1-\frac{x^2}{2}+o(x^2),\sqrt{1+z}=1+\frac{z}{2}+o(z)$,即
\[\sqrt{\cos 2x}= 1-x^2+o(x^2),\quad \sqrt[3]{\cos 3x}= 1-\frac{3x^2}{2}+o(x^2),\quad \sqrt[3]{1+z}=1+\frac{z}{3}+o(z)\]
即
\[
\text{原式}=\lim_{x\rightarrow 0}\frac{1-\left(1-\frac{x^2}{2}+o\left(x^2\right)\right)\left(1-x^2+o\left(x^2\right)\right)\left(1-\frac{3x^2}{2}+o\left(x^2\right)\right)}{x^2}=3
\]
这题方法除泰勒之外,还可简单的等价无穷小,或拆项,洛必达处理
\begin{align*}
\lim_{x\rightarrow 0}\frac{1-\cos x\sqrt{\cos 2x}\sqrt[3]{\cos 3x}}{x^2}&=\lim_{x\rightarrow 0}\left(\frac{1-\cos x}{x^2}+\frac{\cos x\left(1-\sqrt{\cos 2x}\sqrt[3]{\cos 3x}\right)}{x^2}\right)\\
&=\frac{1}{2}+\lim_{x\rightarrow 0}\frac{1-\sqrt{\cos 2x}\sqrt[3]{\cos 3x}}{x^2}\\
&=\frac{1}{2}+\lim_{x\rightarrow 0}\left(\frac{1-\cos\sqrt{2x}}{x^2}+\frac{\cos\sqrt{2x}\left(1-\sqrt[3]{\cos 3x}\right)}{x^2}\right)\\
&=\frac{1}{2}+\lim_{x\rightarrow 0}\left(\frac{1-\sqrt{1+\left(\cos 2x-1\right)}}{x^2}+\frac{1-\sqrt[3]{1+\left(\cos 3x-1\right)}}{x^2}\right)\\
&=\frac{1}{2}+\lim_{x\rightarrow 0}\frac{1-\cos 2x}{2x^2}+\lim_{x\rightarrow 0}\frac{1-\cos 3x}{3x^2}\\
&=3
\end{align*}
\end{solution}
\end{answer}
\ws {解答题}{(\textbf{ 本题满分8分})\\}\\\\
设函数$f(t)$在$t\ne 0$时一阶连续可导,且$f(1)=0$,求函数$f(x^2-y^2)$,使得曲线积分$\displaystyle \int_L{y\left(2-f\left(x^2-y^2\right)\right)}\mathrm {d}x+xf\left(x^2-y^2\right)\mathrm{d}y
$与路径无关,其中$L$为任一不与直线$y=\pm x$相交的分段光滑闭曲线.
\begin{answer}
\begin{solution}
记$\left\{\begin{array}{l}
P\left(x,y\right)=y\left(2-f\left(x^2-y^2\right)\right)\\
Q\left(x,y\right)=x+xf\left(x^2-y^2\right)\\
\end{array}\right. $,于是
\[
\left\{\begin{array}{l}
\vspace{1em}
\frac{\partial P\left(x,y\right)}{\partial y}=2-f\left(x^2-y^2\right)+2y^2f'\left(x^2-y^2\right)\\
\frac{\partial Q\left(x,y\right)}{\partial x}=f\left(x^2+y^2\right)+2x^2f'\left(x^2-y^2\right)\\
\end{array}\right.
\]
由题设可知,积分与路径无关,于是有
\[
\frac{\partial Q\left(x,y\right)}{\partial x}=\frac{\partial P\left(x,y\right)}{\partial y}\Longrightarrow\left(x^2-y^2\right)f'\left(x^2-y^2\right)+f\left(x^2-y^2\right)=1
\]
\hfill\dotfill 5分
记$t=x^2-y^2$,即微分方程
\[
tf'\left(t\right)+f\left(t\right)=1\Leftrightarrow\left(tf\left(y\right)\right)'=1\Rightarrow tf\left(t\right)=y+C
\]
又$f(1)=0$,可得$C=-1\textbf{,}f(t)=1-\frac{1}{t}$,从而
$f\left(x^2-y^2\right)=1-\frac{1}{x^2-y^2}
$
\hfill\dotfill 8分
\end{solution}
\end{answer}
\ws {解答题}{(\textbf{本题满分14分})\\}\\\\
设$f(x)$在区间$[0,1]$上连续,且$1\leq f\left(x\right)\leq3$.证明:
\[
0\leq\int_0^1{f\left(x\right)\mathrm{d}x\int_0^1{\frac{1}{f\left(x\right)}\mathrm{d}x\leq\frac{4}{3}}}
\]
\begin{answer}
\begin{proof}
由 Cauchy-Schwarz 不等式:
\[
\int_0^1{f\left(x\right)\mathrm{d}x\int_0^1{\frac{1}{f\left(x\right)}\mathrm{d}x\geq\left(\int_0^1{\sqrt{f\left(x\right)}\sqrt{\frac{1}{f\left(x\right)}}}\mathrm{d}x\right)}^2}=1
\]
\hfill\dotfill 4分
又由基本不等式得:
\[
\int_0^1{f\left(x\right)\mathrm{d}x\int_0^1{\frac{3}{f\left(x\right)}}\mathrm{d}x}\leq\frac{1}{4}\left(\int_0^1{f\left(x\right)\mathrm{d}x+\int_0^1{\frac{3}{f\left(x\right)}\mathrm{d}x}}\right)^2
\]
再由条件$1\leq f\left(x\right)\leq 3$,有$\left(f\left(x\right)-1\right)\left(f\left(x\right)-3\right)\leq 0$,则
\[
f\left(x\right)+\frac{3}{f\left(x\right)}\leq 4\Rightarrow\int_0^1{\left(f\left(x\right)+\frac{3}{f\left(x\right)}\right)\mathrm{d}x\leq 4}
\]
\hfill\dotfill 10分
即可得
\[
1\le\int_0^1{f\left(x\right)\mathrm{d}x\int_0^1{\frac{1}{f\left(x\right)}\mathrm{d}x\le\frac{4}{3}}}
\]
\hfill\dotfill 14分\\
\end{proof}
\end{answer}
\ws {解答题}{( \textbf{本题满分12分})\\}\\\\
计算三重积分$ \iiint_{\left(V\right)}{\left(x^2+y^2\right)}\mathrm{d}V
$,其中$(V)$是由$x^2+y^2+\left(z-2\right)^2\geq 4$,$x^2+y^2+\left(z-1\right)^2\leq9$及$z\geq0$所围成的空间图形.
\begin{answer}
\begin{solution}
(1)计算打球$(V_1)$的积分,利用球坐标换元,令
\[
\left(V_1\right):\left\{\begin{array}{l}
x=r\sin\varphi\cos\theta ,y=r\sin\varphi\sin\theta ,z-1=r\cos\varphi\\
0\leq r\leq 3,0\leq\varphi\leq\pi ,0\leq\theta\leq 2\pi\\
\end{array}\right.
\]
于是有
\[
\iiint _ { \left( V _ { 1 } \right) } \left( x ^ { 2 } + y ^ { 2 } \right) \mathrm { d } V = \int _ { 0 } ^ { 2 \pi } \mathrm { d } \theta \int _ { 0 } ^ { \pi }\mathrm { d } \varphi \int _ { 0 } ^ { 3 } r ^ { 3 } \sin ^ { 2 } \varphi r ^ { 2 } \sin \varphi = \frac { 8 } { 15 } \cdot 3 ^ { 5 } \pi
\]
\hfill\dotfill 4分
(2)计算小球$(V_2)$的积分,利用球坐标换元,令
\[
\left(V_2\right):\left\{\begin{array}{l}
x=r\sin\varphi\cos\theta ,y=r\sin\varphi\sin\theta ,z-2=r\cos\varphi\\
0\leq r\leq 2,0\leq\varphi\leq\pi ,0\leq\theta\leq 2\pi\\
\end{array}\right.
\]
于是有
\[
\iiint _ { \left( V _ { 2} \right) } \left( x ^ { 2 } + y ^ { 2 } \right) \mathrm { d } V = \int _ { 0 } ^ { 2 \pi } \mathrm { d } \theta \int _ { 0 } ^ { \pi }\mathrm { d } \varphi \int _ { 0 } ^ { 2 } r ^ { 2 } \sin ^ { 2 } \varphi r ^ { 2 } \sin \varphi = \frac { 8 } { 15 } \cdot 2^ { 5 } \pi
\]
\hfill\dotfill 8分
(3)计算大球$z=0$下部分的积分$V_3$,利用球坐标换元,令
\[
\left(V_3\right):\left\{\begin{array}{l}
x=r\cos\theta ,y=r\sin\theta ,1-\sqrt{9-r^2}\leq z\leq 0\\
0\leq r\leq 2\sqrt{2},0\leq\theta\leq 2\pi\\
\end{array}\right.
\]
于是有
\begin{align*}
\iiint_{\left(V_3\right)}{\left(x^2+y^2\right)\mathrm{d}V}&=\iint_{r\leq 2\sqrt{2}}{r\mathrm{d}r}\mathrm{d}\theta\int_{1-\sqrt{9-r^2}}^0{r^2\mathrm{d}z}\\
&=\int_0^{2\pi}{\mathrm{d}\theta}\int_0^{2\sqrt{2}}{r^3\left(\sqrt{9-r^2}-1\right)}\\
&=\left(124-\frac{2}{5}\cdot 3^5+\frac{2}{5}\right)\pi
\end{align*}
综上所述有
\begin{align*}
\iiint _ { ( V ) } \left( x ^ { 2 } + y ^ { 2 } \right) \mathrm { d } V& = \iiint _ { \left( V _ { 1 } \right) } \left( x ^ { 2 } + y ^ { 2 } \right) \mathrm { d } V - \iiint _ { \left( V _ { 2 } \right) } \left( x ^ { 2 } + y ^ { 2 } \right) \mathrm { d } V - \iiint _ { \left( V _ { 3 } \right) } \left( x ^ { 2 } + y ^ { 2 } \right) \mathrm { d } V\\
&= \frac { 8 } { 15 } \cdot 3 ^ { 5 } \pi+\frac { 8 } { 15 } \cdot 2^ { 5 } \pi+\left(124-\frac{2}{5}\cdot 3^5+\frac{2}{5}\right)\pi\\
&=\frac { 256 } { 3 } \pi
\end{align*}
\hfill\dotfill 12分
\end{solution}
\end{answer}
\ws {解答题}{(\textbf{ 本题满分14}分)\\}\\\\
设$f(x,y)$在区域D内可微,且$\sqrt{\left(\frac{\partial f}{\partial x}\right)^2+\left(\frac{\partial f}{\partial y}\right)^2}\leq M$,$A\left(x_1,y_1\right),B\left(x_2,y_2\right)$是D内两点,线段AB包含在D内,证明:
\[
|f\left(x_1,y_1\right)-f\left(x_2,y_2\right)|\leq M|AB|
\]
其中$|AB|$表示线段$AB$的长度.
\begin{answer}
\begin{proof}
作辅助函数
\[
\varphi\left(t\right)=f\left(x_1+t\left(x_2-x_1\right)\cdot y_1+t\left(y_2-y_1\right)\right)
\]
\hfill\dotfill 2分
显然$\varphi(t)$在$[0,1]$可导,根据Lagrange中值定理,存在$c\in(0,1)$,使得
\[
\varphi\left(1\right)-\varphi\left(0\right)=\varphi '\left(c\right)=\frac{\partial f\left(u,v\right)}{\partial u}\left(x_2-x_1\right)+\frac{\partial f\left(u,v\right)}{\partial v}\left(y_2-y_1\right)
\]
\hfill\dotfill 8分
即可得到
\begin{align*}
\left|\varphi\left(1\right)-\varphi\left(0\right)\right|&=\left| f\left(x_2,y_2\right)-f\left(x_1,y_1\right)\right|\\
&=\left|\frac{\partial f\left(u,v\right)}{\partial u}\left(x_2-x_1\right)+\frac{\partial f\left(u,v\right)}{\partial v}\left(y_2-y_1\right)\right|\\
&\leq\sqrt{\left(\frac{\partial f\left(u,v\right)}{\partial u}\right)^2+\left(\frac{\partial f\left(u,v\right)}{\partial v}\right)^2}\cdot\sqrt{\left(x_2-x_1\right)^2+\left(y_2-y_1\right)^2}\\
&\leq M\left| AB\right|
\end{align*}
\hfill\dotfill 14分
\end{proof}
\end{answer}
\ws {解答题}{( \textbf{本题满分14分})\\}\\\\
证明:对于连续函数$f(x)>0$,有
\[
\ln\int_0^1{f\left(x\right)\mathrm{d}x\geq\int_0^1{\ln f\left(x\right)\mathrm{d}x}}
\]
\begin{answer}
\begin{proof}
由定积分定义,将$[0,1]$分$n$等分,可取$\Delta x=\frac{1}{n}$,由“算术平均数$\geq$几何平均数”得:
\[
\frac{1}{n}\sum_{k=1}^n{f\left(\frac{k}{n}\right)\geq\sqrt[n]{f\left(\frac{1}{n}\right)\cdots f\left(\frac{n}{n}\right)}}=\mathrm{exp}{\frac{1}{n}\sum_{k=1}^n{\ln f\left(\frac{k}{n}\right)}}
\]
\hfill\dotfill 4分
\[
\Rightarrow\int_0^1{f\left(x\right)}\mathrm{d}x\geq \mathrm{exp}{\lim_{n\rightarrow\infty}\frac{1}{n}\sum_{k=1}^n{\ln f\left(\frac{k}{n}\right)}}=\mathrm{exp}{\int_0^1{\ln f\left(x\right)}}\mathrm{d}x
\]
\hfill\dotfill 10分
然后两边取对数即证
\[
\ln\int_0^1{f\left(x\right)\mathrm{d}x\geq\int_0^1{\ln f\left(x\right)\mathrm{d}x}}
\]
\hfill\dotfill 14分
或者考虑令$g(x)=\mathrm {ln}x$,则$g'\left(x\right)= \frac{1}{x}$,$g''\left(x\right)=-\frac{1}{x^2}<0$,所以$g(x)$为凹函数,可由琴声不等式定理即证.\\
\end{proof}
\end{answer}
\ws {解答题}{(\textbf{本题满分14分})\\}\\\\
已知${a_k}$,${b_k}$是正数数列,且$b_{k+1}-b_k\geq\delta >0,k=1,2,\cdots $,$\delta$为一切常数,证明:若级数$\sum_{k=1}^{+\infty}{a_k}$收敛,则级数$\sum_{k=1}^{+\infty}{\frac{k\sqrt[k]{\left(a_1a_2\cdots a_k\right)\left(b_1b_2\cdots b_k\right)}}{b_{k+1}b_k}}$收敛.
\begin{answer}
\begin{proof}
令$S_k=\sum_{i=1}^k{a_i}b_i,a_kb_k=S_k-S_{k-1},S_0=0,a_k=\frac{s_k-S_{k-1}}{b_k},k=1,2,\cdots $
\hfill\dotfill 4分
\begin{align*}
\sum_{k=1}^N{a}_k
& =\sum_{k=1}^N{\frac{S_k-S_{k-1}}{b_k}}=\sum_{k=1}^{N-1}{\left(\frac{S_k}{b_k}-\frac{S_k}{b_{k+1}}\right)}+\frac{S_N}{b_N}\\
&=\sum_{k=1}^{N-1}{\frac{b_{k+1}-b_k}{b_kb_{k+1}}}S_k+\frac{S_N}{b_N}\geq\sum_{k=1}^{N-1}{\frac{\delta}{b_kb_{k+1}}}S_k
\end{align*}
所以$\sum_{k=1}^{\infty}{\frac{S_k}{b_kb_{k+1}}}$收敛.
\hfill\dotfill 10分
由算术-几何平均不等式得
\[
\sqrt[k]{\left(a_1a_2\cdots a_k\right)\left(b_1b_2\cdots b_k\right)}\leq\frac{a_1b_1+\cdots +a_kb_k}{k}=\frac{S_k}{k}
\]
\[
\sum_{k=1}^{\infty}{\frac{k\sqrt[k]{\left(a_1a_2\cdots a_k\right)\left(b_1b_2\cdots b_k\right)}}{b_{k+1}b_k}}\leq\sum_{k=1}^{\infty}{\frac{S_k}{b_kb_{k+1}}}
\]
故结论成立.
\hfill\dotfill 14分
\end{proof}
\end{answer}
\mbox{}
%试卷正文结束
\end{document}